LSAT and Law School Admissions Forum

Get expert LSAT preparation and law school admissions advice from PowerScore Test Preparation.

 Administrator
PowerScore Staff
  • PowerScore Staff
  • Posts: 8916
  • Joined: Feb 02, 2011
|
#40587
Complete Question Explanation
(The complete setup for this game can be found here: lsat/viewtopic.php?t=11817)

The correct answer choice is (A)

The question stem establishes that T is given at H, which can be linked to the fifth rule:
PT70 -Game_#4_#21_diagram 1.png
The first result of this assignment is that the first lecture must be on R. Only R or T can be the first lecture, and because the first lecture is given at G, T can no longer be the first lecture. Thus, R must be the first lecture:
PT70 -Game_#4_#21_diagram 2.png
The second inference that results from this assignment is that with both S and T known to be given at H, the other three lectures—O, P, and R—must each be given at G. Some of those assignments were already known (P, and then R was inferred to be at G), but now O is know to be at G as well. Thus, the location of the four unassigned lectures is known, and they are all involved in the fourth and fifth rules:
PT70 -Game_#4_#21_diagram 3.png
Of course, either S or T must be fourth since those are the only two possibilities for H, and that creates two solutions for this question:

Solution #1: S is fourth
PT70 -Game_#4_#21_diagram 4.png
Solution #2: T is fourth
PT70 -Game_#4_#21_diagram 5.png
In the second solution, O can be third and be given at G, and thus answer choice (A) could be true and is correct.
You do not have the required permissions to view the files attached to this post.
 angelsfan0055
  • Posts: 39
  • Joined: Feb 26, 2021
|
#85720
Hi there, for whatever reason I'm not understanding from the explanations why Terns can't be at Howard Hall. The explanation mentions the fifth rule but doesn't that say that Petrel is given in Gladwyn Hall?
 Adam Tyson
PowerScore Staff
  • PowerScore Staff
  • Posts: 5153
  • Joined: Apr 14, 2011
|
#85778
The lecture on Terns IS given in Howard Auditorium, angelsfan0055! That's what the local restriction in this question required. But that can't happen third, because if it was third then we would be unable to satisfy both of the sequence rules. Try drawing it out and you'll find that when HT is third, HS has to be fourth because there are only two H's in the game). Then where do we put GP, which has to be after T, and O, which has to be after HS? We've run out of room! And there's nothing that can go second in this scenario, either. Thus, the HT block cannot go third. Give it a try and you'll see!

Get the most out of your LSAT Prep Plus subscription.

Analyze and track your performance with our Testing and Analytics Package.